LSAT and Law School Admissions Forum

Get expert LSAT preparation and law school admissions advice from PowerScore Test Preparation.

 Administrator
PowerScore Staff
  • PowerScore Staff
  • Posts: 8917
  • Joined: Feb 02, 2011
|
#24696
Complete Question Explanation

Strengthen—PR. The correct answer choice is (B)

Here we learn about the dangers of lead paint, whose effective removal requires experienced contractors. In spite of this fact, most homeowners choose to undertake such removal on their own. The author of this stimulus argues that Homeowner’s Journal should run an article on how to reduce the risks associated.

The question stem requires us to find the principle that would strengthen the case for such an article.

Answer choice (A): This principle would weigh against running the article, because if this principle is applied, the paint removal should never be done by untrained homeowners.

Answer choice (B): This is the correct answer choice. If this principle is applied, then the magazine should indeed run an article on minimizing risk, because this is a case in which homeowners refuse to change their behavior in spite of their awareness of the risks associated.

Answer choice (C): Since the project discussed in the stimulus does entail substantial risks, this principle would not strengthen the case for running the article, and this answer choice cannot be correct.

Answer choice (D): The risk discussed in the stimulus is risk to one’s self, not to other people, so this principle is not applicable and does not strengthen the argument for running the lead-removal safety article.

Answer choice (E): This is not about whether or not to discourage homeowners from taking on this dangerous project, and the referenced homeowners are not necessarily competent for this particular task. Thus, this principle would not bolster the case for printing the article, and this answer choice cannot be correct.
 yournoona
  • Posts: 18
  • Joined: Mar 13, 2020
|
#74720
Hello,
Although i got the correct answer, I have trouble understanding as to why the question is termed as Strengthen PR rather than Justify the conclusion type. Could you please help me understand why the first is correct as opposed to second.

Regards
Yournoona
 Adam Tyson
PowerScore Staff
  • PowerScore Staff
  • Posts: 5153
  • Joined: Apr 14, 2011
|
#74732
It all comes down to "most", yournoona. A Justify question never uses a qualifier like "most" or "helps", because Justify is about going all the way to 100% proof. It's the difference between me carrying most of your groceries into the house for you, and me doing the entire job on my own. Strengthen answers do at least some of the work, but they don't have to do it all (they may, but they don't have to). Justify answers do more than just help, and do more than just "most" of the work.

Get the most out of your LSAT Prep Plus subscription.

Analyze and track your performance with our Testing and Analytics Package.